生產功能

如何從生產函式中推導出公司的成本函式?

  • August 7, 2015

我最近學習瞭如何使用拉格朗日乘數的方法解決以下類型的問題:

給定一個具有效用函式的消費者 $ u(x,y) $ , 財富 $ w $ , 價格 $ p =(p_x,p_y) $ , 預算約束 $ w = xp_x + yp_y $ . 求需求函式 $ x $ 和 $ y $ . 我還學習瞭如何通過將這些需求函式插入我的預算約束並重寫以給出成本函式來找到成本函式。

但這些案件都涉及 $ w $ 並以某種方式對該預算進行預算。

問題設置:

假設一家公司是一個價格接受者,該產品具有市場價格 $ p $ . 假設企業有生產函式 $ F(K,L) $ . 推導企業成本函式。

我的問題:

我應該如何解決這個問題?

我與解決問題相關的想法/想法:

我沒有財富 $ w $ 儘管我想我可以假設任何購買的公司都有一些預算。我之前做的所有問題都涉及兩種商品,但這涉及一種商品。也許我可以因此有一個預算約束 $ w = qp $ 在哪裡 $ q $ 是數量和 $ p $ 是給予價格接受公司的價格。但是我看到的拉格朗日乘數問題涉及兩個變數,這只是一個的函式。

評論:

我使用的具體生產函式是 $ F(K,L) = \sqrt{k} + \sqrt{l} $ . 我只包括這個,以防它是相關的。我想自己解決問題。

“我沒有得到財富 $ w $ 儘管我想我可以假設任何正在採購的公司都有一些預算。”

,這正是公司的基本微觀經濟理論與微觀經濟消費者理論不同的地方:公司不是受預算限制。原因在於,這一基本理論主要涉及“長期”觀點,甚至更好的是“計劃觀點”。所以我們假設支付費用所需的金額將來自銷售,因為公司不會虧本進入生產(還要記住,這是一個確定性的設置,沒有不確定性)。營運資金方面的考慮(通常首先您必須實際支付費用然後實際收取收入的事實),並沒有進入長期的視野,這是一種短期現象。此外,在長期或計劃方法中,沒有固定成本,所有因素都是可變的。

現在,解決公司優化問題的“成本最小化”方法是利潤最大化設置的另一種行為假設,它在許多實際案例中非常相關:主要是為了滿足需求而存在的公用事業,以及他們的動機不是使利潤最大化——而是在有效利用始終稀缺的資源的情況下,根據需求確定 的給定產出水平的成本最小化。

但是,與市場相比,價格接受公司的規模太小,更接近於成本最小化而不是利潤最大化,因為該公司並沒有真正控制其生產(除非通過直接決策向下) .

在上述兩種情況下,都會出現一個外生變數:產出水平本身。因此,我們通過將輸出水平視為“恆定”或更好來解決問題,我們針對任何給定的輸出水平解決它,並且我們獲得的解決方案將輸出水平作為其組成部分之一。

所以

$$ \min_{K,L} C\equiv rK + wL \ s.t. F(K,L) = \bar Q $$ 與拉格朗日

$$ \Lambda = rK + wL +\lambda[\bar Q - F(K,L)] $$ 一階條件是

$$ r = \lambda F_K,;;; w=\lambda F_L \tag{1} $$ 在最佳情況下,它給出了,

$$ rK + wL = C = \lambda\big(F_KK + F_L L) \tag{2} $$ 現在假設生產函式在某種程度上是齊次的 $ h $ (不一定是一級同質的,即表現出“規模收益不變”,而是同質的——而你的是同質的 $ h=1/2 $ .)。從歐拉定理的齊次函式 $ h $ 我們有

$$ F_KK + F_L L = hF(K,L) = h\bar Q \tag{3} $$ 給定初始問題的約束,最後一個等式成立。插入 $ (3) $ 進入 $ (2) $ 我們獲得

$$ C = \lambda h \bar Q $$ 乘數 $ \lambda $ 是最優邊際成本,記為 $ C’(\bar Q) $ ,所以我們到達

$$ C = C’(\bar Q)\cdot (h\bar Q) \implies C’(\bar Q) + [(-1/h\bar Q)]\cdot C =0 $$ 這是一個有解的簡單齊次微分方程

$$ C = A\cdot \exp\left{-\int(-1/h\bar Q) {\rm d}\bar Q \right} = A\cdot \exp\left{(1/h)\ln \bar Q\right} $$ $$ \implies C^* = A\cdot (\bar Q)^{1/h} \tag{4} $$ 對於一些常數 $ A >0 $ . 為了完成解決方案,我們需要表達感興趣的對象, $ C^* $ ,就外生實體而言: $ r,w,\bar Q $ . 為此,得出最佳邊際成本(等於乘數)

$$ (4) \implies \lambda^* = (1/h)A(\bar Q)^{1/h-1} \tag{5} $$ 插入 $ (5) $ 進入我們有的一階條件

$$ r = (1/h)A(\bar Q)^{1/h-1} F_K,;;; w=(1/h)A(\bar Q)^{1/h-1} F_L \tag{6} $$ 是時候使用生產函式的具體函式形式了

$$ F(K,L) = K^{1/2} + L^{1/2} \implies, F_K = \frac 12 K^{-1/2},;; F_L = \frac 12 L^{-1/2} \tag{7} $$ 惰化 $ (7) $ 進入 $ (6) $ 和…一起 $ h=1/2 $ 我們得到,經過操縱,

$$ rK = \frac {A^2}{r}(\bar Q)^2,;; wL = \frac {A^2}{w}(\bar Q)^2 \tag{8} $$ 將兩者相加以獲得成本函式的替代表達式

$$ rK+wL = C^* = A(\bar Q)^2\cdot \left[\frac Ar + \frac Aw\right] \tag{9} $$ 但是插入 $ h=1/2 $ 進入 $ (4) $ , 我們也有

$$ C^* = A(\bar Q)^2 \tag{10} $$ 所以 $$ (9),(10) \implies A(\bar Q)^2\cdot \left[\frac Ar + \frac Aw\right] = A(\bar Q)^2 $$ $$ \implies \frac Ar + \frac Aw = 1 \implies A = \frac {wr}{w+r} \tag {11} $$ 插入 $ (11) $ 進入 $ (4) $ 我們得出結論

$$ C^* = \frac {wr}{w+r}\cdot (\bar Q)^2 \tag{12} $$ 三件事:

**A)**驗證二階條件是否適用於所有這些,以確實導致最優成本函式。

**B)**解決具有相同生產函式的無約束利潤最大化問題,將產出價格標準化為 $ p=1 $ (即處理外生價格, $ w,r $ 以實際值表示),以驗證它是否會導致與它一致的成本水平 $ (12) $ .

**C)**如果您對預算約束下的公司理論感興趣,相關論文是 Lee, H., & Chambers, RG (1986)。美國農業的支出限制和利潤最大化。美國農業經濟學雜誌,68(4),857-865。

引用自:https://economics.stackexchange.com/questions/3020